Proving rigorously the supremum of a set

$\begingroup$ The first thing would be to say what $A$ is a subset of. Like that, it does not have an upper bound, so no sup, or $+\infty$ if you will. In $[0,2)\cup [3,+\infty)$, the sup is $3$. But in $\mathbb$, of course, the sup is $2$. $\endgroup$

Commented May 9, 2013 at 16:32 $\begingroup$ Okay, I will edit it. Thanks. Is my proof correct? $\endgroup$ Commented May 9, 2013 at 16:35

$\begingroup$ So we consider $A=[0,2)\subseteq \mathbb$. Step 1: $A$ is bounded above by $2$. So there is a finite sup and $\sup A\leq 2$. Step 2: if $M$ is a bound of $A$ from above, then $x\leq M$ for every $0\leq x\leq M$ for every $n\geq 1$. So $2\leq M$ to the limit. In particular, $2\leq \sup A$. $\endgroup$

Commented May 9, 2013 at 16:38

$\begingroup$ That is quicker indeed - but is my proof okay? I suppose I could have deleted the bit about Archimedean since I didn't have to use such an $n$. $\endgroup$

Commented May 9, 2013 at 16:43

$\begingroup$ I see at least one mistake: $M-1>0$ implies $2(M-1)>0$, ont $2(M-1)>2$. And if you wanted the latter to hold, you would need $M-1>1$, that is $M>2$. So I'm afraid you are in a circular reasoning. $\endgroup$

Commented May 9, 2013 at 16:48

3 Answers 3

$\begingroup$

I think there's a way more simple and intuitive proof.

First, as you observed, it is obvious that 2 is an upper bound. Now, to prove it is the supremum. Assume that $M$ is the supremum and $M

Of course, $M>2$ is trivially impossible since $2$ is an upper bound as well and thus any $M$ bigger than $2$ cannot be a supremum.

But now, assume it is not, that is $x\geq2$.

Then, $\frac+M\geq2$. Multiplying both sides by $2$, we get

Now, as for the simplicity of this proof, I have written a lot for clarity and in case you are a beginner on this subject. This can be summarized in $2$ lines, but this is for clarity. I hope this helps, and you must soon learn to find the shortest and more intuitive way. Good luck.

answered Apr 14, 2015 at 18:30 Hasan Saad Hasan Saad 4,593 1 1 gold badge 18 18 silver badges 32 32 bronze badges $\begingroup$ but what is x representing here? $\endgroup$ Commented May 7, 2020 at 3:59 $\begingroup$ @JosephRock The midpoint between $2$ and $M$. $\endgroup$ Commented May 11, 2020 at 3:36 $\begingroup$ how can the midpoint be larger than the supremum? $\endgroup$ Commented May 11, 2020 at 3:39 $\begingroup$ Now it's not clear why one shoudl we even consider the midpoint $\endgroup$ Commented Aug 23 at 15:12 $\begingroup$

Let $a\lt 2$ is $\sup A$ .

Therefore, $a=2-b$ , where $b\gt 0$ .

We can get some $n\in \mathbb \,|\, 0 \lt \frac1n \lt b$

$\exists c\in Q\cap A\,|\,2\gt c\gt 2-(\frac1n)\gt 0$ .

So, $\,2\gt c \gt 2-(\frac1n)\gt (2-b)=a$ .

$0\lt c \lt 2 \implies c\in A$ , and also $c\gt a$

i.e. $a$ is not even an upper bound of $A$ .

$\implies$ the set of upper bounds of $A$ is $\$ and $x\ge 2\>=S$ and the least member of $S$ is $2$ .

So, $2$ is the least upper bound of $A$ .

4,590 2 2 gold badges 16 16 silver badges 36 36 bronze badges answered Apr 14, 2015 at 18:17 803 3 3 gold badges 8 8 silver badges 18 18 bronze badges

$\begingroup$ In short, this means: "Whatever number you take smaller than 2, there will be a bigger than it number in A, so the number you chose is not an upper bound." Am I right? This answer is more clear to me because Hasan defines x in an unfamiliar way for me. $\endgroup$

Commented Oct 5, 2018 at 21:45 $\begingroup$

I find the some of the existing proofs either magic or non sensical, hence I decided to write my own proof and submit to this important question.

Note : I use the definition of supremum as smallest upper bound.

We have that the supremum can't be greater than $2$ very easily by the definition which says that it is the "smallest upper bound". Now, the supremum can be in the interval $(-\infty,2]$ .

I claim that it can also not be less than or equal $0$ . Before that, we first proof $\frac12$ is an element of $[0,2)$ . We have that, $ 0

If it were that the supremum of the set, $s$ were less than or equal $0$ , we'd have that:

Which is a contradiction with the part of the definition of the supremum which says that the supremum must be greater than any number inside the set (an upper bound).

Hence the supremum is in the interval $(0,2]$ . Let us suppose the supremum $s$ is smaller than $2$ , then

That is, the midpoint between the supremum and two is a number that is in the set (field property of real number) and again bigger than it. But, this is also an element in the given set, which means $s$ can't be a supremum as there exist an element greater than it which is also in the set.